LSAT and Law School Admissions Forum

Get expert LSAT preparation and law school admissions advice from PowerScore Test Preparation.

 Administrator
PowerScore Staff
  • PowerScore Staff
  • Posts: 8916
  • Joined: Feb 02, 2011
|
#32015
Complete Question Explanation
(The complete setup for this game can be found here: lsat/viewtopic.php?t=15241)

The correct answer choice is (D)

If exactly one ranger is assigned to area 1, we cannot resort to Template 1, because in that template there are at least two rangers (J and K) assigned to area 1. Therefore, Template 2 provides the only feasible solution. In that template, O must be assigned to area 3—a prephrase that validates answer choice (D).

While it is unnecessary to develop the full range of solutions in order to confirm that answer choice (D) is correct, such an approach is illustrated below for instructional purposes:

If exactly one ranger is assigned to area 1, L cannot be that ranger. This is because L must be assigned to the same area as at least one other ranger (K or M) in compliance with the third rule. Therefore, L is either assigned to area 2 with K, or else it is assigned to area 3 with M and O. This limits the solutions presented in Template 2 as follows:
PT79_Game_#2_#8_diagram 1.png
Answer choice (A): This answer choice is incorrect, because J need not be assigned to area 1.

Answer choice (B): This answer choice is incorrect, because K could also be assigned to area 1.

Answer choice (C): This answer choice is incorrect, because L could be assigned to area 2.

Answer choice (D): This is the correct answer choice.

Answer choice (E): This answer choice is incorrect, because P can be assigned to either area 2 or to area 3.
You do not have the required permissions to view the files attached to this post.

Get the most out of your LSAT Prep Plus subscription.

Analyze and track your performance with our Testing and Analytics Package.